Question

Consider the following series. ∞ 1 n4 n = 1 (a) Use the sum of the...

Consider the following series. ∞ 1 n4 n = 1 (a) Use the sum of the first 10 terms to estimate the sum of the given series. (Round the answer to six decimal places.) s10 = 0.082036 Incorrect: Your answer is incorrect. (b) Improve this estimate using the following inequalities with n = 10. (Round your answers to six decimal places.) sn + ∞ f(x) dx n + 1 ≤ s ≤ sn + ∞ f(x) dx n ≤ s ≤ (c) Using the Remainder Estimate for the Integral Test, find a value of n that will ensure that the error in the approximation s ≈ sn is less than 0.00001. n > −32 n > 5 n > 15 n > 0 n > 32

Homework Answers

Know the answer?
Your Answer:

Post as a guest

Your Name:

What's your source?

Earn Coins

Coins can be redeemed for fabulous gifts.

Not the answer you're looking for?
Ask your own homework help question
Similar Questions
1. Use a power series to approximate the definite integral, I, to six decimal places. 0.4...
1. Use a power series to approximate the definite integral, I, to six decimal places. 0.4 to 0, (x5 / 1 + x6 ) dx 2. Find a power series representation for the function. (Give your power series representation centered at x = 0.) f(x) = ln(9 − x). Determine the radius of convergence, R. I already found the first part to be x is 1/n(x/9)^n but can't find R
(a) Find the Riemann sum for f(x) = 3 sin(x), 0 ≤ x ≤ 3π/2, with...
(a) Find the Riemann sum for f(x) = 3 sin(x), 0 ≤ x ≤ 3π/2, with six terms, taking the sample points to be right endpoints. (Round your answers to six decimal places.) R6 = (b) Repeat part (a) with midpoints as the sample points. M6 = Express the limit as a definite integral on the given interval. lim n → ∞ n 7xi* + (xi*)2 Δx, [3, 8] i = 1 8 dx 3
Consider the following function. f(x) = ln(1 + 2x),    a = 1,    n = 3,    0.8 ≤ x ≤...
Consider the following function. f(x) = ln(1 + 2x),    a = 1,    n = 3,    0.8 ≤ x ≤ 1.2 (a) Approximate f by a Taylor polynomial with degree n at the number a. T3(x) = (b) Use Taylor's Inequality to estimate the accuracy of the approximation f(x) ≈ Tn(x) when x lies in the given interval. (Round your answer to six decimal places.) |R3(x)| ≤ (c) Check your result in part (b) by graphing |Rn(x)|.
Approximate the sum of the series correct to four decimal places, ((-1)^n-1*n^2)/12^n
Approximate the sum of the series correct to four decimal places, ((-1)^n-1*n^2)/12^n
Given the alternating series:    n=2∞(-1)^n/ln(n) (7 pts) Determine if the series converge absolutely.    (Use the fact...
Given the alternating series:    n=2∞(-1)^n/ln(n) (7 pts) Determine if the series converge absolutely.    (Use the fact that: ln n < n ) (7 pts) Determine if the series converge conditionally. (7 pts) Estimate the sum of the infinite series using the first 4 terms in the series and estimate the error. (7 pts) How many terms should we use to approximate the sum of the infinite series in question, if we want the error to be less than 0.5?
Use a power series to approximate the definite integral to 4 decimal places: from 0 to...
Use a power series to approximate the definite integral to 4 decimal places: from 0 to 1/2 (x^2)(e^(-x^2) dx. Find power series of e^-x^2. and the value of the integral (how many terms needed)
Use a graphing calculator Riemann Sum (found here) to find the following Riemann sums. f(x) =...
Use a graphing calculator Riemann Sum (found here) to find the following Riemann sums. f(x) = 2/x   from  a = 1  to  b = 5 (a) Calculate the Riemann sum for the function for the following values of n: 10, 100, and 1000. Use left, right, and midpoint rectangles, making a table of the answers, rounded to three decimal places. n Left Midpoint Right 10 100 1000 (b) Find the exact value of the area under the curve by evaluating an appropriate definite...
Given the alternating series: sigma(2 to infinity): (-1)^n / ln n Determine if the series converge...
Given the alternating series: sigma(2 to infinity): (-1)^n / ln n Determine if the series converge absolutely.    (Use the fact that: ln n < n) Determine if the series converge conditionally. (Estimate the sum of the infinite series using the first 4 terms in the series and estimate the error. How many terms should we use to approximate the sum of the infinite series in question, if we want the error to be less than 0.5?
(a) Use any test to show that the following series is convergent. X∞ n=1 (−1)n n...
(a) Use any test to show that the following series is convergent. X∞ n=1 (−1)n n 2 + 1 5 n + 1 (b) Find the minimum number of terms of the series that we need so that the estimated sum has an |error| < 0.001.
Use Simpson's Rule with n = 10 to estimate the arc length of the curve. Compare...
Use Simpson's Rule with n = 10 to estimate the arc length of the curve. Compare your answer with the value of the integral produced by your calculator. (Round your answer to six decimal places.) y = sec(x) + 1, 0 ≤ x ≤ π/3